Showing that two rings are not isomorphic

  • Thread starter Thread starter Mr Davis 97
  • Start date Start date
  • Tags Tags
    Rings
Click For Summary
SUMMARY

The rings R = ℤ/3 × ℤ/3 and S = (ℤ/3)[x]/(x²+1) are not isomorphic. The reasoning is based on the fact that S is isomorphic to (ℤ/3)[i], where i satisfies i² = 2. Since (ℤ/3)[i] is not isomorphic to ℤ/3 × ℤ/3, the conclusion is definitive that R and S cannot be isomorphic.

PREREQUISITES
  • Understanding of ring theory and isomorphisms
  • Familiarity with finite fields, specifically ℤ/3
  • Knowledge of polynomial rings and quotient structures
  • Basic concepts of complex numbers in modular arithmetic
NEXT STEPS
  • Study the properties of finite fields and their extensions
  • Learn about polynomial rings and their quotient structures
  • Explore the concept of isomorphism in abstract algebra
  • Investigate the implications of non-isomorphic rings in algebraic structures
USEFUL FOR

Students of abstract algebra, mathematicians interested in ring theory, and educators teaching concepts of isomorphism and finite fields.

Mr Davis 97
Messages
1,461
Reaction score
44

Homework Statement


Are the two rings ##R = \mathbb{Z}/3 \times \mathbb{Z}/3## and ##S = (\mathbb{Z}/3)[x]/(x^2+1)## isomorphic or not?

Homework Equations

The Attempt at a Solution


I think that they are not isomorphic. I think this because it seems to be the case that ##(\mathbb{Z}/3)[x]/(x^2+1) \cong (\mathbb{Z}/3)[ i ]##, but that ##(\mathbb{Z}/3)[ i ] \not \cong \mathbb{Z}/3 \times \mathbb{Z}/3##. Am I on the right track or is this wrong?
 
Last edited:
Physics news on Phys.org
Sounds good. You have to explain what ##i## is, namely ##i^2=2## and perhaps you can show what ##\mathbb{Z}_3## is isomorphic to.
 
Last edited:
  • Like
Likes Mr Davis 97
Question: A clock's minute hand has length 4 and its hour hand has length 3. What is the distance between the tips at the moment when it is increasing most rapidly?(Putnam Exam Question) Answer: Making assumption that both the hands moves at constant angular velocities, the answer is ## \sqrt{7} .## But don't you think this assumption is somewhat doubtful and wrong?

Similar threads

  • · Replies 4 ·
Replies
4
Views
2K
  • · Replies 7 ·
Replies
7
Views
2K
Replies
2
Views
1K
  • · Replies 3 ·
Replies
3
Views
2K
  • · Replies 1 ·
Replies
1
Views
1K
  • · Replies 5 ·
Replies
5
Views
2K
  • · Replies 1 ·
Replies
1
Views
2K
Replies
3
Views
2K
  • · Replies 1 ·
Replies
1
Views
2K
Replies
3
Views
1K